A survey found that 50% of teenagers prefer to watch a movie at a theater over other viewing options. You want to know the estimated probability that three out of four randomly chosen teenagers do not prefer watching a movie at a theater. How could you design a simulation for this situation?

Answers

Answer 1

Answer:

7m tall convert this into imperial measurement


Related Questions

HI PLEASE ANSWER THIS QUESTION THANK YOU!
(NO TROLLS PLEASE!)

Answers

Not 100% sure but it might be B

Population 422 increasing by 10 people per year. Write an equation to determine the population (y) after any number of years (x).

Answers

Answer:

y = 10x + 422

Step-by-step explanation:

HELP ME!! FIND X AND Y.

Answers

Answer:

x = 26

y = 72°

Step-by-step explanation:

Finding x:Alternates angles are equal.

(4x - 16)° = (3x + 10)°

Subtract 3x to both sides

4x - 3x - 16 = 10

x - 16 = 10

Add 16 to both sides

x = 10 + 16

x = 26

Finding y:Angles on a straight lines add up to 180 degrees.

(4x - 16)° + y° = 180°

4x - 16 + y = 180

4(26) - 16 + y = 180

104 - 16 + y = 180

88 + y = 180

Subtract 88 to both sides

y = 180 - 88

y = 72°

[tex]\rule[225]{225}{2}[/tex]

which statements could be an interpretation of the graph x intercept or y intercept

Answers

Answer:

If you attach the answers i can help you better

Step-by-step explanation:

Reflect shape A in the x-axis.
I have 20 min to complete this

Answers

Answer:

Step-by-step explanation:

The length of a rectangle is twice its breadth. If its length is decreased half of the 10 cm and the breadth is increased by half of the 10 cm cm, the area of the rectangle is increased by 5 sq cm, more than 70 sq cm. Find the length of the rectangle.

Also....any armies, stays and blinkss?

Answers

Since the length of this rectangle is twice its breadth, the length is equal to 40 cm.

How to calculate the area of a rectangle?

Mathematically, the area of a rectangle can be calculated by using this formula:

Area = Length × Width

Based on the information provided, we have:

Its length is decreased half of the 10 cm ⇒ (length - 5)Its breadth is increased by half of the 10 cm ⇒ (breadth + 5)Its area is increased by 75 cm ⇒ (area + 5)

Substituting the parameters into the formula, we have:

LB + 75 = (L - 5)(B + 5)

2B² + 75 = (2B - 5)(2B + 5)

2B² + 75 = 2B(B + 5) - 5(B + 5)

2B² + 75 = 2B² + 10B - 5B - 25

5B = 75 + 25

5B = 100

B = 100/20

B = 20 cm.

For the length, we have:

L = 2B

L = 2 × 20

L = 40 cm.

Read more on rectangle here: https://brainly.com/question/25292087

#SPJ1

(-2/3)^3 (Evaluate)

A: 8/9
B: 6/9
C: -8/27
D: 8/27

Answers

Answer:

C

Step-by-step explanation:

using the rule of exponents

[tex](\frac{a}{b}) ^{m}[/tex] = [tex]\frac{a^{m} }{b^{m} }[/tex] , then

[tex](\frac{-2}{3}) ^{3}[/tex]

= [tex]\frac{(-2)^{3} }{3^3}[/tex]

= [tex]\frac{-8}{27}[/tex]

= - [tex]\frac{8}{27}[/tex]

Power with a natural exponent.

Definition:

[tex]a^2=a\cdot a\\\\a^3=a\cdot a\cdot a\\\\a^4=a\cdot a\cdot a\cdot a\\\vdots\\a^n=\underbrace{a\cdot a\cdot a\cdot...\cdot a}_{n}[/tex]

also

[tex]a^1=a\\\\a^0=1[/tex]

We have

[tex]\left(-\dfrac{2}{3}\right)^3=\left(-\dfrac{2}{3}\right)\cdot\left(-\dfrac{2}{3}\right)\cdot\left(-\dfrac{2}{3}\right)=-\dfrac{2\cdot2\cdot2}{3\cdot3\cdot3}=-\dfrac{8}{27}[/tex]

[tex]\huge\boxed{C:\ -\dfrac{8}{27}}[/tex]

Solve by elimination
show your work
y = 3x + 10
y = 2x - 8
​im giving brainlyest

Answers

Answer:

[tex]x=-18, y=-44[/tex]

Step-by-step explanation:

[tex]y=3x+10\\y=2x-8\\\\3x+10=y\\2x-8=y\\3x-y=-10\\2x-y=8\\\\-2(3x-y=-10)\\3(2x-y=8)\\\\-6x+2y=20\\6x-3y=24\\-y=44\\y=-44\\-44=3x+10\\-54=3x\\x=\frac{-54}{3} \\x=-18\\\\x=-18, y=-44\\[/tex]

CHECK

[tex]-44=3(-18)+10\\-44=-54+10\\-44=-44\\\\-44=2(-18)-8\\-44=-36-8\\-44=-44[/tex]

help the question is on the image.​

Answers

Answer:

[tex]\boxed {tan\theta = \frac{2}{3}}[/tex]

Step-by-step explanation:

Let's derive a formula :

⇒ Area (triangle) × 3 = Area (square)

⇒ 1/2(bh) × 3 = a²

∴ But a = h (shown in diagram)

⇒ 3/2(bh) = h²

⇒ h = 3b/2

Now, taking tan θ :

⇒ tan θ = b/h

⇒ tan θ = b/(3b/2)

⇒ tan θ = 2b/3b

tan θ = 2/3

SOS help please ignore the answers I put they are wrong

Answers

The conversion to logarithm function will be that log3 27 = 3.

How to calculate the log?

From the information given, we are to convert the exponential equation to logarithm function. This will be:

log 3 27

This will be log 3 3³

= 3 × log 3^3

= 3 × 1

= 3

Therefore, x = 3.

Learn more about logarithms on:

brainly.com/question/25710806

#SPJ1

Let f(r) = 42 - 5, find f(3).

Answers

F (x) = 4x - 5
F (3) just means substituting x with 3
For further please ask in question

The length of a rectangle is units greater than twice its width. If its width is w, which expression gives the perimeter of the rectangle in terms of w?
A.
2(w) + w
B.
w + w
C.
3w +
D.
6w + 5

Answers

The expression that gives the perimeter of the rectangle in terms of w is 6w+5 units.

Given that, the length of the rectangle is [tex]\frac{5}{2}[/tex] units greater than twice its width.

We need to determine the expression that gives the perimeter of the rectangle in terms of w.

Given, width = w units, then length= 2w + [tex]\frac{5}{2}[/tex] units.

What is the perimeter of the rectangle?

The perimeter of the rectangle is the sum of all the sides of the rectangle.

That is, perimeter of a rectangle = 2 (length+width).

Now, perimeter of a rectangle = 2 (2w + [tex]\frac{5}{2}[/tex]+w)=6w+5 units.

Hence, the expression that gives the perimeter of the rectangle in terms of w is 6w+5 units.

To learn more about the perimeter of a rectangle visit:

https://brainly.com/question/15287805.

#SPJ1

Which transformations could have taken place? Select two options.

Answers

The two options are R₀ 270 , R₀ 90 , Option C and D .

The complete question is attached in the image.

What are the two types of Transformations ?

There are two types of transformations namely , Rigid Transformation and Non- Rigid Transformation

The rigid transformations do not change the shape and size of the preimage .

The non Rigid transformation do not change the shape but change the size of the preimage.

The rule rotating the origin , 270 degree is

(x,y) ----> (y , -x)

as (x,y) is (0,5)

Therefore after 270 degree transformation

(y,-x) is (5 , 0)

So, the clockwise rotation is 90 degree about origin

(x,y) ----> (y , -x)

as (x,y) is (0,5)

Therefore after 90 degree transformation

(y,-x) is (5 , 0)

Therefore the two options are R₀ 270 , R₀ 90 , Option C and D .

To know more about Types of Transformations

https://brainly.com/question/5020733

#SPJ1

After how many seconds does the object reach its maximum height? 2 seconds 3 seconds 6 seconds 9 seconds

Answers

Answer:

3 seconds

proofi took the test

The object reaches its maximum height after 3 seconds.

The correct option is 3 seconds.

To find the time at which the object reaches its maximum height, we need to determine the vertex of the parabolic function[tex]h(t) = -16t^2 + 96t + 6[/tex] . The vertex form of a parabola is given by [tex]h(t) = a(t - h)^2 + k[/tex], where (h, k) represents the vertex.

In this case, a = -16, so the vertex is given by t = -b/2a. Plugging in the values, we get t = -96/(2 x (-16)) = 96/32 = 3 seconds.

Therefore, the object reaches its maximum height after 3 seconds.

Option B, 3 seconds, is the correct answer.

To learn more about parabolas;

https://brainly.com/question/21685473

#SPJ2

The complete Question:

The function [tex]h(t) = -16t^2 + 96t + 6[/tex] represents an object projected into the air from a cannon. The maximum height reached by the object is 150 feet.

After how many seconds does the object reach its maximum height?

A. 2 seconds

B. 3 seconds

C. 6 seconds

D. 9 seconds

URGENT!!! PLEASE HELP!!! Find x and y.

Answers

Answer:

x is 48 and y is 100

Step-by-step explanation:

Angle y = 100 degree

Angle x = 48 degree

1. In a class of 24, 3 students were absent, What fraction of class is present

2. Karen taped a television show that lasted 1h 30mins. On the same tape she recorded another 2 shows that were 5/6h each. What time was still left on the 4h tape.

3. Steve ordered 12 lengths of timber, each 6m long. He cut 6 lengths into 1 1/2m pieces and the other 6 lengths into 3/4 pieces A. How many pieces together is this? B. Was there any waste?

Answers

Problem 1

Answer:  7/8 of the class is present

Reason:

24 people total with 3 absent. So 24-3 = 21 are present

21/24 = (7*3)/(8*3) = 7/8

==========================================================

Problem 2

Answer:  50 minutes left on the tape.

Work Shown:

1 hour = 60 min

(5/6)*(1 hour) = (5/6)*(60 min)

5/6 of an hour = 50 min

1 hr + 30 min = 60 min + 30 min = 90 min

The first show is 90 minutes. Add on two more shows that are 50 min each to get 90+50+50 = 190 minutes used up

4 hours = 4*60 = 240 min

240 - 190 = 50 min

There are 50 minutes left on the tape.

==========================================================

Problem 3

Answer:  72 pieces; not enough info to calculate the waste

Explanation:

1 1/2 = 1 + 1/2 = 1 + 0.5 = 1.5

6/(1.5) = 4

Each 6 meter board is cut into 4 equal pieces each 1.5 meters long

There are 6 boards this applies to which means so far that's 6*4 = 24 pieces

Now to the other 6 boards.

3/4 = 0.75

6/(0.75) = 8

Each of the other 6 boards are cut into 8 equal pieces of 0.75 meters each

That's another 6*8 = 48 pieces

In total there are 24+48 = 72 pieces

When cutting into wood, there always is a small sliver that is discarded. While small, these slivers do add up. So yes there is some waste. Unfortunately, we don't have enough info to calculate. We would need to know the width of the saw blade.

what is the cos? trigonometry ​

Answers

Answer:

Cos, a abreviation of cosine is used in trigonometry, and it represents the ratio of the hyptenuse and the adjacent side of a right triangle (a triangle with a right/90 degree angle). The formula for evaluating cosine/cos is the length of the adjacent side of the triangle divided by length of the hypotenuse of the triangle. To be clear, the hypotenuse is the side of the triangle the 90 degrees is facing, and the adjacent side is the side across the hypotenuse.

Step-by-step explanation:

Answer:

A trigonometric function that for an acute angle is the ratio between the leg adjacent to the angle when it is considered part of a right triangle and the hypotenuse.

What is cos law?

The cos law or cosine law defines the relationship between the sides and the angles of the triangle. This law is also known as the law of cosine or cosine rule. According to this law,

c2 = a2 + b2 − 2ab cos α

Where a, b and c are the sides of the triangle.

Which expression is equal to the polynomial
below?
2x + 5x38x - 20
Ox³(2x + 5) + 4(2x - 5)
Ox³(2x + 5)- 4(2x + 5)
Ox³(2x-5)-4(2x - 5)
Ox³(2x + 5) + 4(2x + 5)

Answers

The expression 2x⁴ + 5x³ - 8x - 20 can be express as follows;

x³(2x + 5) - 4(2x + 5)

How to factorise an expression?

2x⁴ + 5x³ - 8x - 20

Therefore,

2x⁴ + 5x³ - 8x - 20

x³(2x + 5) - 4(2x + 5)

Hence, the expression 2x⁴ + 5x³ - 8x - 20 can be express as follows:

2x⁴ + 5x³ - 8x - 20 = x³(2x + 5) - 4(2x + 5)

learn more on expression here: https://brainly.com/question/26228081

#SPJ1

if 13 sin θ=12 cos θ, find the value

Answers

From the given information, we have

[tex]13\sin(\theta) = 12\cos(\theta) \implies \dfrac{\sin(\theta)}{\cos(\theta)} = \dfrac{12}{13}[/tex]

In the expression of interest, divide through everything by [tex]\cos^2(\theta)[/tex] to get

[tex]\dfrac{2\sin(\theta)\cos(\theta)}{\cos^2(\theta) - \sin^2(\theta)} = \dfrac{2\frac{\sin(\theta)}{\cos(\theta)}}{1 - \frac{\sin^2(\theta)}{\cos^2(\theta)}}[/tex]

Then plugging in the given info, we get

[tex]\dfrac{2\sin(\theta)\cos(\theta)}{\cos^2(\theta) - \sin^2(\theta)} = \dfrac{2\times \frac{12}{13}}{1 - \left(\frac{12}{13}\right)^2} = \boxed{\dfrac{312}{25}}[/tex]

the slope of the line is 4. which of the following is the point-slope form of the line

Answers

The point slope form of line is y+4 = -4( x+3)

What is point slope of form?

The point slope form is used to find the equation of the straight line which is inclined at a given angle to the x-axis and passes through a given point.

Given:

m=4

We know the general form is

y - b = -4(x - a)

So, from the graph attached below

b= -4, a= -3

So, y-(-4)= -4(x-(-3))

y+4 = -4( x+3)

Hence,  point slope form of line is y+4 = -4( x+3).

Learn more about tis concept here:

https://brainly.com/question/13436232

#SPJ1

Pls help quickly!!!

Find the coordinates of the vertex of the graph of the function.
y=3x^2-4x+3

Answers

Answer:

vertex = ( [tex]\frac{2}{3}[/tex], [tex]\frac{8}{3}[/tex] )

Step-by-step explanation:

given the equation of a parabola in standard form

y = ax² + bx + c ( a ≠ 0 )

then the x- coordinate of the vertex is

x = - [tex]\frac{b}{2a}[/tex]

y = 3x² - 4x + 3 ← is in standard form

with a = 3 , b = - 4 , then

x = - [tex]\frac{-4}{6}[/tex] = [tex]\frac{2}{3}[/tex]

substitute x = [tex]\frac{2}{3}[/tex] into the equation for corresponding y- coordinate of vertex

y = 3([tex]\frac{2}{3}[/tex] )² - 4([tex]\frac{2}{3}[/tex] ) + 3

  = 3([tex]\frac{4}{9}[/tex] ) - [tex]\frac{8}{3}[/tex] + 3

  = [tex]\frac{4}{3}[/tex] - [tex]\frac{8}{3}[/tex] + [tex]\frac{12}{3}[/tex]

  = [tex]\frac{8}{3}[/tex]

vertex = ( [tex]\frac{2}{3}[/tex], [tex]\frac{8}{3}[/tex] )

Vertex ( 2/3 , 8/3 )

The ratio of players to soccer balls at practice sessions is 5:2. How many soccer balls are needed for 20 players?​

Answers

Answer:

8 soccer balls

Step-by-step explanation:

Since the ratio of players to soccer balls is 5:2, we can conclude that:

Players = 5 peopleSoccer balls = 2 balls

The question asks to find how many soccer balls are needed for 20 players.

Multiply both players and soccer balls by 4, this will get:

Players = 20 peopleSoccer balls = 8 balls

Henceforth, 8 soccer balls are needed for 20 players.

Selena is preparing for her eighth-grade graduation party. she must keep within the budget set by her parents. which is the best price for her to purchase ice cream? a. $3.99/ 24 oz carton b. $4.80/ one-quart carton c. $11.00 / one gallon tub d. $49.60/ five gallon tub

Answers

Answer:

Basically find the unit rate by doing 3.99/24 = $0.16625/1oz

1 quart = 32 ounces so 4.8/32 = $0.15/1oz

1 gallon = 153.722 ounces so 11/153.722 = 0.07

49.6/768.61 = 0.06

Step-by-step explanation:
D.

Please help with this question

Answers

Answer:

4

Step-by-step explanation:

the second column and the second row so, the answer is 4

help its TIMED

WHICH OF THE FOLLOWING MAY BE TRUE ABOUT f(x)

Answers

All three are possible.

The table tells us that for certain x and y, we have f(x) < f(y) if x < y. There's also no evidence that f(x) = f(y) for some points x ≠ y in the interval, so f(x) could easily be monontonically increasing. (I is true)

If we assume f(x) is strictly increasing (which does not contradict the given table) on the entire interval, we have f'(x) > 0. A positive function can be strictly increasing. (II is true)

A function is concave up on an interval if its derivative is increasing on that interval. This follows from statement (II) if it happens to be true. (III is true)

(E)

Jaime makes the following claim.
An angle with a measure of π/3 radians in a circle with a radius of 3 inches is smaller than an angle with a measure of π/3 radians in a circle with a radius of 6 inches. This is because the radian measure is based on the length of the radius of the circle and a radius of 3 inches is smaller than a radius of 6 inches.

1. Review Jaime’s statement and what you know about angles and radians.
a. Explain whether you agree with Jaime’s statement or not.
b. Provide examples to support your whether you agree or disagree.

Answers

Jaime is incorrect, the angle does not depend on the radius of the circles.

Is Jaime correct?

Remember that an angle that defines an arc on a circle, does not depend on the radius of the circle.

So, if we have an angle with a measure of π/3 radians in a circle with a radius of 3 inches and an angle with a measure of π/3 radians in a circle with a radius of 6 inches, these two angles are exactly the same thing.

The radius of the circle only has an impact on the length of the arc defined by the angle.

So Jaime is clearly incorrect.

If you want to learn more about angles:

https://brainly.com/question/17972372

#SPJ1

How to subtract negative number from 1

Answers

when you subtract a negative number from any platitude number the equation changes to addition so you would add the value of the negative number to 1.

To subtract a negative number from one you will need to set it up like this

1- (-x) x equals the negative number you're talking about

so we use the method KCC (keep change change)

So we KEEP the positive one, CHANGE the subtraction sign in to a plus sign "+" then CHANGE the negative number to a positive

lets say the negative number is 2

1-(-2)

1+ (+2)

1+2 = 3

Fraction:
23
100
Decimal: 0.23
Percent: 30 %

Answers

Answer: Below is the correct data.

Fraction: [tex]\frac{23}{100}[/tex]

Decimal: 0.23

Percent: 23%

I hope this helps! Pls mark brainliest!! :)

Suppose ∠A and ∠B are complements, while ∠B and ∠C are supplements. If m∠A=3x+17, m∠B=55−x, and m∠C=67(x/3−1), then m∠A=44, m∠B=46, and m∠C=134. TRUE OR FALSE? Please show your work.

Answers

Answer:

TRUE

Step-by-step explanation:

Angle <A and angle <B are complementary, which means their sum must be = 90

Angle <B and angle <C are supplementary which means their sum must be = 180

Now let's see

m∠A=3x+17, m∠B=55−x, and m∠C=67(x/3−1)

first find the sum of A and B

3x + 17 + 55 - x = 90 add like terms

2x + 72 = 90 subtract 72 from both sides

2x = 18 divide both sides by 2

x = 9

TRUE OR FALSE is asked for

m∠A=44, m∠B=46, and m∠C=134

to find that we just replace c with 9 with every expression given for angles

m<A = 3x + 17

m<A = 3*9 + 17 = 44

m<B =  55 - x

m<B = 55 - 9 = 46

m<C = 134, we don't really need to calculate one by one since the angle are complementary and supplementary.

The answer is TRUE

The statement is true if A and ∠B are complements, while ∠B and ∠C are supplements.

What is an angle?

When two lines or rays converge at the same point, the measurement between them is called a "Angle."

We have:

∠A and ∠B are complements, while ∠B and ∠C are supplements.

∠A + ∠B = 90 degree

3x + 17 + 55 - x = 90

2x = 18

x = 9

Angle ∠B and angle ∠C are supplementary:

∠B + ∠C = 180

55 -x + 67(x/3  - 1) = 180

x = 9

Put the value of x and find angles A, B, and C.

m∠A = 44

m∠B = 46

m∠C = 134

∠A + ∠B = 90

∠B + ∠C = 180

The above statement is true.

Thus, the statement is true if A and ∠B are complements, while ∠B and ∠C are supplements.

Learn more about the angle here:

brainly.com/question/7116550

#SPJ

I already worked on most of my question, although, I am just curious about how you would go about finding the fourth power of the square root of 50. Can someone please explain this in detail?

Answers

Answer:

(50^1/2)^4

Step-by-step explanation:

basically if I am comprehending your question correctly, the square root of 50 can also be written as 50^1/2

If you put 50^1/2 to the 4th power, it basically becomes (50^1/2)^4

Other Questions
SAfe is based on four primary bodies of knowledge# A company just starting business made the following four inventory purchases in June: Date Number of units purchased Total cost June 1 120 units $ 420 June 10 150 units 600 June 15 150 units 570 June 28 200 units 270 $1860 A physical count of merchandise inventory on June 30 reveals that there are 210 units on hand. Using the average-cost method, the amount allocated to the ending inventory on June 30 is Describe two characteristics found in the Mona Lisa that portrays the Classicalinfluence. D adhesion and cohesion of wale24. Choose the correct statement that will make the following sentence true. "If a plant cell is put into a solution with avery high concentration of solute,....A water will leave the vacuole.Bthe cell will be turgid (the vacuole will be filled with water).C the cell membrane will be held out by the pressure of water against the cell wall.D. water will no longer enter the cell. Find the value of this expression if x = -7.x +5x+1Enter the correct answer. Suppose you hold a portfolio of two stocks in the healthcare industry. The future outcomes for these stocks depend mainly on the next healthcare bill to be passed by Congress. The possible outcomes and returns are: Outcome Probability Return for Stock A Return for Stock B 1=Republican Bill 0.8 12% -5% 2=Democratic Bill 0.2 3% 20% What is the expected return and a standard deviation of a portfolio that has 50% of its value in stock A and 50% in stock B? Find the value of the lettered angle b Seven-year-old hannah can arrange 10 buttons in order from smallest to largest. Her understanding of _____ allows her to accomplish this. PLEASE HELP RNNN HELPPPPPPPPPPPP Potential output is not purely a physical measure based on the number of workers and existing factories because: 12 times a number is decreased by 10% of the number. Write an expression. what goal did the magma carta accomplish Select the correct answer from each drop-down menu. juan picks up a steel utensil and immediately drops it because its extremely hot. how does the reflex arc work to protect juan from a worse burn? in a reflex arc, the neuron carries the action potential from the . Which of these best explains how solid waste can harm wildlife?A. Aquatic animals drown after getting tangled in fishing line. B. Bacteria are introduced to break down hydrocarbons in an oil spill. C. The mercury contained in fish builds up in animals that eat the fish. D. An increased growth in microorganisms results in oxygen depletion. Match the verbal expression (term) with its algebraic expression (definition).Match Term DefinitiondSome number increased by two A) a 2cA variable decreased by two B) y2eProduct of an unknown value and two C) x 22Quotient of some number and two D) b + 2bAn unknown value squared E) 2z A triangle has two sides of lenghts 8 and 10. what value could the length of the third sird side be? COMPLETE THE FOLLOWING EQUATION USING APPROPRIATE SIGN ( +,-,x,) IN THE BOXES GIVEN:REMEMBER TO USE (DMAS) How many moles of sulfur are in 80 grams of sulfur?A. 1.5 molesB. 2.5 molesC. 1 moleD. 2 moles Find the distance between the points (2, 3) and (4, 7). how to write a story ending with the words:"Never will I do that again"